LSAT and Law School Admissions Forum

Get expert LSAT preparation and law school admissions advice from PowerScore Test Preparation.

User avatar
 Dave Killoran
PowerScore Staff
  • PowerScore Staff
  • Posts: 5853
  • Joined: Mar 25, 2011
|
#49519
Complete Question Explanation
(The complete setup for this game can be found here: lsat/viewtopic.php?t=17097)

The correct answer choice is (D)

Answer choice (A): From the last rule we know that when T is added to class 1 then Z must also be added to class 1. This answer violates that rule, and thus cannot be true.

Answer choice (B): The second rule stipulates that S must be added to class 3. Because S is in class 2 in this answer, this answer is incorrect.

Answer choice (C): The fourth rule indicates that V and Z cannot be added to the same class. This answer is therefore incorrect because V and Z both appear in class 2.

Answer choice (D): This is the correct answer choice.

Answer choice (E): The third rule states that W and Y cannot be added to the same class. This answer is therefore incorrect because W and Y both appear in class 2.

The information from the hypothetical discovered in this question (namely that V can be added to class 2) can then be used to eliminate answer choices (A), (B), and (C) in question #2. The same hypothetical can be used to eliminate answer choices (C) and (E) in question #3.

Get the most out of your LSAT Prep Plus subscription.

Analyze and track your performance with our Testing and Analytics Package.